vollst. Induktion, Ungleichung, rekursive Folge

Neue Frage »

torsti3208 Auf diesen Beitrag antworten »
vollst. Induktion, Ungleichung, rekursive Folge
Meine Frage:
Sei eine reelle Zahl. Betrachten Sie die rekursiv definierte Folge , gegeben durch und
Zeigen Sie, dass gilt:


Meine Ideen:
Okay Induktionsanfang sollte



sein.
Im Induktionsschritt habe ich n+1 einfach aus der Aufgabenstellung genommen, und so umgestellt:


Nun weiß ich im Beweis nicht weiter. Überlegungen die ich mir gemacht habe:
Wenn dann muss sein. Es ist ja echt größer 0, daher kann bei Teilen durch 2 nicht 0 oder weniger rauskommen.
Außerdem da dann muss
In der Summe sollte es also > 0 sein.
Leider weiß ich nicht wie ich das in die Gleichung importieren soll, oder wie ich es sonst zeigen kann.
Vielen Dank für evtl. Hilfe!
jimmyt Auf diesen Beitrag antworten »
RE: vollst. Induktion, Ungleichung, rekursive Folge
Zitat:
Original von torsti3208
Meine Frage:
Sei eine reelle Zahl. Betrachten Sie die rekursiv definierte Folge , gegeben durch und
Zeigen Sie, dass gilt:


Meine Ideen:
Okay Induktionsanfang sollte



sein.
Im Induktionsschritt habe ich n+1 einfach aus der Aufgabenstellung genommen, und so umgestellt:

...


Naja, wenn deine Induktionsannahme für ein diese ist,
dann ist die Induktionsbehauptung .
Dann machst du den Induktionsschritt mit n -> n+1. Und schau dir mal den Nenner genauer an. smile
torsti3208 Auf diesen Beitrag antworten »
RE: vollst. Induktion, Ungleichung, rekursive Folge
Aber ist nicht meine Induktionsannahme? Das soll ich ja zeigen. Dann wäre noch für n=n+1 die Induktionsbehauptung

Oder verstehe ich da etwas falsch?
jimmyt Auf diesen Beitrag antworten »

Naja, das hier

Zitat:
Original von torsti3208
Zeigen Sie, dass gilt:



ist, meiner bescheidenen Meinung nach, die Aufgabenstellung.
Du sollst beweisen, daß das für alle gilt.

Induktionsannahme für ein :



Induktionsbehauptung für n+1:



Und den Induktionschritt würde ich mit der Behauptung anfangen, und dann die I.V. einsetzen. smile


Um diese rekursive Folge vlt. besser zu verstehen, setze doch einfach mal die ersten 2-3 Zahlen für ein.
Also geg. ist .

Dann für:

n=0:

n=1:

n>=2:
torsti3208 Auf diesen Beitrag antworten »

Okay das hat mir für das Verständnis der Folge wirklich weiter geholfen.
Aber wenn ich die beiden im Term durch ersetze , erhalte ich ein Ungetüm welches ich nicht zu berechnen im Stande bin Hammer
Induktionsannahme = Induktionsvorraussetzung, das habe ich richtig verstanden oder?
Oh man, an der Aufgabe hab ich bis jetzt schon Stunden verbracht..
jimmyt Auf diesen Beitrag antworten »

Ja, mit Induktionsannahme meine ich Induktionsvoraussetzung.

Und zu deinem Ungetüm:
Ja, habe ich auch erhalten. Hat sich mir auch schon vorgestellt.
Aber mußt du es wirklich berechnen?
Oder geht es vielleicht auch mit Schlußfolgerungen aufgrund des Induktionsanfangs und der I.V.? smile
 
 
torsti3208 Auf diesen Beitrag antworten »

Ich denke nicht das ich das berechnen kann / muss. Wenn durch Schlussfolgerungen der Beweis gültig ist, sollte das auch reichen. Was auf eine meiner Ursprungfragen zurückführt - kann ich das einfach als Text formulieren wie ich es in meinem ersten Post gemacht habe, oder gibt es da eine formale Ausdrucksweise?
Grautvornix Auf diesen Beitrag antworten »

Zitat:
Original von torsti3208
Oh man, an der Aufgabe hab ich bis jetzt schon Stunden verbracht..

Das ist bedauerlich. Insbesondere weil in deinem Eingangspost doch schon alles Wesentliche steht:

1. lt Vor. ist x>0

2. IA: (n=0) gilt nach Def.

3. IV: Sei

4. IS: Mit und sind natürlich auch und und somit auch deren Summe größer als 0.

Fertig!
torsti3208 Auf diesen Beitrag antworten »

Jetzt bin ich ein klein wenig verwirrt - jimmyt nannte den Term für als Induktionsvorraussetzung und nicht , wie ich in meinem ersten Post annahm. Dementsprechend IS anstelle von .
Was ist nun richtig? Sollte in meinem ersten Post die richtige Lösung bereits enthalten sein, mag mir jemand evtl. einen Strick reichen?
jimmyt Auf diesen Beitrag antworten »

Zitat:
Original von torsti3208
Jetzt bin ich ein klein wenig verwirrt - jimmyt nannte den Term für als Induktionsvorraussetzung und nicht , wie ich in meinem ersten Post annahm. Dementsprechend IS anstelle von .
Was ist nun richtig? Sollte in meinem ersten Post die richtige Lösung bereits enthalten sein, mag mir jemand evtl. einen Strick reichen?


Bitte kein Strick. geschockt

Grautvornix hat aus meiner Sicht recht, das hätte mir auch gleich auffallen müssen.
Aber das du jetzt verwirrt bist kann ich nachvollziehen.

Viele Wege führen nach Rom. Eleganter ist Grautvornix Vorschlag.
Beim Induktionsschritt wird für ein beliebiges n angenommen, das es stimmt. Wenn es für n stimmt, dann muss es auch für n+1 gelten. Also Induktionschritt n -> n+1.
Wenn es für den Induktionsanfang klappt, für ein beliebiges n und auch für n+1 gilt, dann ist es für alle n bewiesen.
Sowohl für den Schritt von n->n+1, als auch für n+1->n+2 muss es funktionieren, wenn es für alle n gelten soll.

Sorry, vielleicht habe ich dich wirklich verwirrt.
Und der Weg von Grautvornix ist eleganter, aber deswegen ist der andere nicht falsch.
Und rechnen mußt du bei beiden Varianten nicht viel. smile
Grautvornix Auf diesen Beitrag antworten »

Zitat:
Original von torsti3208
Was ist nun richtig?


Beides ist richtig!
Es muss nur jeweils alles zusammen passen.

Wenn Du als IV annimmst, dass die Beh. für bereits bewiesen sei, dann ist im IS, mittels dieser IV, die Gültigkeit der Behauptung für n+1 zu folgern.

Meiner Ansicht nach hat dein Eingangspost also bereits alles Wesentliche enthalten - sorry! Augenzwinkern
torsti3208 Auf diesen Beitrag antworten »

Ich danke euch beiden!
Wenn wir uns schon eingerbeitet haben und noch jemand Zeit/Lust hat, selbe Vorraussetzungen:

Zitat:
Sei eine reelle Zahl. Betrachten Sie die rekursiv definierte Folge , gegeben durch und



Nur diesmal soll sein Augenzwinkern Mithilfe dem Hinweis, zuerst zu zeigen, das Quadrat einer reellen Zahl ist.

____________________

Folgende Überlegung:



Da als Abbildung von verstanden wird, und ist, und außerdem jede reelle Zahl >= 0 eine Quadratwurzel hat, ist auch Quadrat einer reellen Zahl.

Zu :
Da im Körper der reellen Zahlen keine Quadratwurzeln für negative Zahlen existieren, darf x nicht größer als sein, sont wäre widerlegt. Also gilt

Passt das so, oder habe ich irgendwo einen Fehler gemacht?
Danke.
jimmyt Auf diesen Beitrag antworten »

Die Aufgabe verstehe ich nicht. Wenn du eben bewiesen hast mit vollständiger Induktion,
daß immer gilt, dann gilt auch .
Grautvornix Auf diesen Beitrag antworten »

Offenbar gilt für alle




Daraus folgt




Umformen dieser Ungleichung (unter Beachtung von ) liefert dann:

torsti3208 Auf diesen Beitrag antworten »

Ich verstehe weder, warum die 2. Gleichung aus der ersten folgt, noch wie man von der Gleichung auf die 3. Gleichung kommt.
MatheIstLustig Auf diesen Beitrag antworten »

ich gebe mal zwei Tipps, da die beiden Anderen zur Zeit nicht antworten:

2. Ungleichung folgt aus 1.Ungleichung:
löse die beiden Klammern auf und fasse zusammen.

2. Ungleichung folgt 3. Ungleichung

so umformen, dass links nur noch x steht;
dann rechte Seite zu umformen
torsti3208 Auf diesen Beitrag antworten »

Hallo, leider bin ich nicht auf den Trichter gekommen. die 4a^2x bleiben für mich Zauberei.
Aber danke auf jeden Fall an euch, vll reichen die restlichen Aufgaben ja zum Bestehen der Übung.
Neue Frage »
Antworten »



Verwandte Themen

Die Beliebtesten »
Die Größten »
Die Neuesten »